Morin 3.7 -- Block sliding sideways on an inclined plane

  • #1
masteralien
36
2
Thread moved from the technical forums to the schoolwork forums
TL;DR Summary: In Morin 3.7 sliding sideways on a plane I used a completely different method than he did and got the correct answer is my method right

The problem statement is as follows
I split up the friction force into x and y components derived a diff eq for v_y in terms of v_x then took the limit as v_x goes to 0

IMG_3052.jpeg

My solution was this

$$m\ddot{x} = -\mu mg\cos(\theta) \cos(\varphi)$$
$$\\m\ddot{y}= -mg\sin(\theta)-\mu mg\cos(\theta)\sin(\varphi)$$
$$\\\mu = \tan(\theta)$$
$$\\v_x=\dot{x}$$
$$\\v_y=\dot{y}$$
where $$\varphi$$ is the angle the velocity vector makes
$$\\\dot{v}_x=-g\sin(\theta) \frac{v_x}{\sqrt{v_x^2 + v_y^2}}$$
$$\\\dot{v}_y=-g\sin(\theta)-g\sin(\theta) \frac{v_y}{\sqrt{v_x^2 + v_y^2}}$$

At long times v_x’ will be 0 so we can set final v_x = 0 as setting v_x’=0 means v_x will be 0

I then divided to get

$$\frac{dv_y}{dv_x}=\frac{v_y}{v_x} + \sqrt{1+\left(\frac{v_y}{v_x}\right)^2}$$

I substituted v_y/v_x = u and with some algebra setting the lower bound of v_x as V and upper as v_x to take the limit later and for v_y I took the upper bound as v_y and lower as 0 get


$$\frac{v_x^2}{V}=\sqrt{v_x^2+v_y^2}+v_y$$

$$\frac{v_x^4}{V^2}-\frac{2v_y v_x^2}{V}+v_y^2=v_x^2+v_y^2$$

Then get
$$\frac{v_x^2}{V^2}-\frac{2v_y}{V}=1$$

Finally taking v_x goes to 0

$$v_y = -\frac{V}{2}$$

With the final speed being
$$V_f=\frac{V}{2}$$
I want to know if this solution is correct as this gives the correct answer

This is the book’s solution

IMG_3054.jpeg
 
Last edited:
Physics news on Phys.org
  • #2
masteralien said:
I substituted v_y/v_x = u and with some algebra setting the lower bound of v_x as V and upper as v_x to take the limit later and for v_y I took the upper bound as v_y and lower as 0 get

$$\frac{v_x^2}{V}=\sqrt{v_x^2+v_y^2}+v_y$$
Unfortunately, that does not give the right answer if you plug in ##v_x=0##.
Please post the omitted algebra.
 
Last edited:
  • #3
Its right below the algebra
 
  • #4
haruspex said:
Unfortunately, that does not give the right answer if you plug in ##v_x=0##.
Please post the omitted algebra.
The steps are right below. I even checked with a graphing calculator that as v_x approaches 0 v_y approaches -V/2
 
  • #5
masteralien said:
The steps are right below.
You wrote
masteralien said:
and with some algebra
Can you post that?
masteralien said:
I even checked with a graphing calculator that as v_x approaches 0 v_y approaches -V/2
But if you substitute ##v_x=0## in the equation I quoted you get ##v_y=0##, no?
 
  • #6
haruspex said:
You wrote

Can you post that?

But if you substitute ##v_x=0## in the equation I quoted you get ##v_y=0##, no?
No my steps are right below I subtracted v_y and squared both sides then divided by v_x^2 then took the limit. You need to take the limit as it approaches as this for long times its a bit like terminal velocity.
 
  • #7
masteralien said:
No my steps are right below
I am asking you to show how you got from
masteralien said:
$$\frac{dv_y}{dv_x}=\frac{v_y}{v_x} + \sqrt{1+\left(\frac{v_y}{v_x}\right)^2}$$
to
masteralien said:
$$\frac{v_x^2}{V}=\sqrt{v_x^2+v_y^2}+v_y$$
I do not see that anywhere.

masteralien said:
I subtracted v_y and squared both sides
Squaring introduces extra solutions. Specifically here, you are now solving
$$\frac{v_x^2}{V}=\pm\sqrt{v_x^2+v_y^2}+v_y$$
If we substitute ##v_x=0## in the extra solution that introduces:
$$\frac{v_x^2}{V}=-\sqrt{v_x^2+v_y^2}+v_y$$
we get 0=0, so this spurious solution is the one that, through taking the limit, arrives at ##v_y=V/2##.

$$\frac{v_x^2}{V}=\sqrt{v_x^2+v_y^2}+v_y$$ cannot be right because ##v_x=0## implies ##v_y=0##, whether by taking limits or simply plugging in. It seems likely that it should have been $$\frac{v_x^2}{V}=-\sqrt{v_x^2+v_y^2}+v_y$$.
You can avoid introducing extra solutions by dividing that through by ##v_x##, applying the binomial expansion and taking limits.
 
  • #8
Also v_x would never truly reach 0 just get very close and its best to express the solution a bit differently. Also I have plugged in my equation the first into desmos to check and as you decrease v_x it goes to -V/2 but never set v_x=0
$$v_y=v_x\sinh(\ln(\frac{v_x}{V}))$$
Taking x to 0
=$$V x\sinh(\ln(x))$$
=$$V x/2 (x-1/x)= V /2 (x^2-1)$$
=$$-V/2$$ as x goes to 0
 
  • #9
haruspex said:
I am asking you to show how you got from

to

I do not see that anywhere.


Squaring introduces extra solutions. Specifically here, you are now solving
$$\frac{v_x^2}{V}=\pm\sqrt{v_x^2+v_y^2}+v_y$$
If we substitute ##v_x=0## in the extra solution that introduces:
$$\frac{v_x^2}{V}=-\sqrt{v_x^2+v_y^2}+v_y$$
we get 0=0, so this spurious solution is the one that, through taking the limit, arrives at ##v_y=V/2##.

$$\frac{v_x^2}{V}=\sqrt{v_x^2+v_y^2}+v_y$$ cannot be right because ##v_x=0## implies ##v_y=0##, whether by taking limits or simply plugging in. It seems likely that it should have been $$\frac{v_x^2}{V}=-\sqrt{v_x^2+v_y^2}+v_y$$.
You can avoid introducing extra solutions by dividing that through by ##v_x##, applying the binomial expansion and taking limits.
Also is my solution physically sound like is the setup logical physically
 
  • #10
haruspex said:
I am asking you to show how you got from

to

I do not see that anywhere.


Squaring introduces extra solutions. Specifically here, you are now solving
$$\frac{v_x^2}{V}=\pm\sqrt{v_x^2+v_y^2}+v_y$$
If we substitute ##v_x=0## in the extra solution that introduces:
$$\frac{v_x^2}{V}=-\sqrt{v_x^2+v_y^2}+v_y$$
we get 0=0, so this spurious solution is the one that, through taking the limit, arrives at ##v_y=V/2##.

$$\frac{v_x^2}{V}=\sqrt{v_x^2+v_y^2}+v_y$$ cannot be right because ##v_x=0## implies ##v_y=0##, whether by taking limits or simply plugging in. It seems likely that it should have been $$\frac{v_x^2}{V}=-\sqrt{v_x^2+v_y^2}+v_y$$.
You can avoid introducing extra solutions by dividing that through by ##v_x##, applying the binomial expansion and taking limits.
You would have to solve for v_y
 

Similar threads

  • Introductory Physics Homework Help
Replies
3
Views
846
  • Introductory Physics Homework Help
Replies
12
Views
2K
  • Introductory Physics Homework Help
Replies
15
Views
392
Replies
2
Views
82
  • Introductory Physics Homework Help
Replies
14
Views
2K
  • Introductory Physics Homework Help
2
Replies
53
Views
4K
  • Introductory Physics Homework Help
Replies
3
Views
1K
  • Introductory Physics Homework Help
2
Replies
55
Views
3K
  • Introductory Physics Homework Help
Replies
8
Views
4K
Back
Top